Đến nội dung

Hình ảnh

$\sum \frac{a}{a+bc} \ge \frac{3}{2}$


  • Please log in to reply
Chủ đề này có 1 trả lời

#1
chuyenamsbest

chuyenamsbest

    Lính mới

  • Thành viên mới
  • 4 Bài viết

Cho $a+b+c=3$.CMR $\sum \frac{a}{a+bc} \ge \frac{3}{2}$


Bài viết đã được chỉnh sửa nội dung bởi perfectstrong: 14-09-2021 - 13:16
Tiêu đề + LaTeX


#2
PDF

PDF

    Trung sĩ

  • Thành viên
  • 197 Bài viết

Cho $a+b+c=3$.CMR $\sum \frac{a}{a+bc} \ge \frac{3}{2}$

Không mất tính tổng quát, giả sử $a$ là số lớn nhất trong ba số $a,b,c$.

Khi đó ta có $a\geq 1$. Áp dụng bất đẳng thức Cauchy-Schwarz và bất đẳng thức AM-GM ta có

$$\frac{b}{b+ac}+\frac{c}{c+ab}\geq \frac{(b+c)^{2}}{b^{2}+c^{2}+2abc}=\frac{1}{1+\dfrac{2bc(a-1)}{(b+c)^{2}}}\geq \frac{1}{1+\dfrac{a-1}{2}}=\frac{2}{a+1},$$

$$\frac{a}{a+bc}\geq \frac{a}{a+\dfrac{(b+c)^{2}}{4}}=\frac{4a}{4a+(3-a)^{2}}.$$

Ta chỉ cần chứng minh $$\frac{4a}{4a+(3-a)^{2}}+\frac{2}{a+1}\geq \frac{3}{2},$$

hay là $$\frac{3(3-a)(a-1)^{2}}{2(a^{2}-2a+9)(a+1)}\geq 0,$$

đúng do $a\leq 3$.

Đẳng thức xảy ra chỉ khi $a=b=c=1$. $\square$






0 người đang xem chủ đề

0 thành viên, 0 khách, 0 thành viên ẩn danh